Projectile motion (1 Viewer)

zeek

Member
Joined
Sep 29, 2005
Messages
549
Location
ummmmm
Gender
Male
HSC
2006
Can someone please post their solution for the projectile motion?! I have no idea how they got L2 =2V2t2(1-sin@) -2aVtcos@ + a2
All i got was L2 =V2t2cos2 -2aVtcos@ + a2 after doing Δx=a-Vtcos@ and then squaring both sides. :confused:
 

abcd9146

Member
Joined
Dec 4, 2005
Messages
107
Gender
Male
HSC
2006
ur missing something... i think, well it was kinda the same/different to what i had in the exam, but my memory is all fuzzy @__@

well what i did was use the distance formula and then i eventually got it right... i think pythagoras also worked, but i didn't try that...

you can try it again xD
 

nanini

New Member
Joined
Jan 27, 2006
Messages
9
Gender
Female
HSC
2006
i had no idea about that question! i still don't see how you do it?!
 

kakamwamwa

New Member
Joined
Jul 11, 2006
Messages
1
Gender
Female
HSC
2006
That was a ridiculous question for question 6!
don't take my word for it but i THINK you had to make the
cos^2x into 1-sin^2x which then became (1+sinx)(1-sinx)
and then somehow the 1+sinx could cancel out...

mmm thats all i can remember. its all a big blur :p

Does anyone else know the solution?
 

ianc

physics is phun!
Joined
Nov 7, 2005
Messages
619
Location
on the train commuting to/from UNSW...
Gender
Male
HSC
2006
Yeah basically what you had was something like this:

Therefore for the distance, you use pythagoras

L2 = (y2 - y1)2 + (a - x1)2

where x1 = vtcos(theta)
y1 = Vtsin(theta) - (1/2)gt2
y2 = Vt - (1/2)gt2

if you didn't remember to subtract the y2, your result will be the same as zeek's. initially i forgot as well...

for ii)

you differentiate L in terms of t, to get a massively ugly thing for dL/dt. You then solve for dL/dt=0 and t will come out to be the thing in the question

Then you substitute t into the formula for L to get that distance (more nasty algebra)

then for iii)

You need to find the time range in which Particle one is ascending

So you differentiate to get y'= Vsin(theta) - gt

Then you find the time when maximum height occurs (ie y'=0)

then you say that this time must be > t=acos(theta)/(2v blah blah)

and you will get V > that thing
 
Last edited:

Users Who Are Viewing This Thread (Users: 0, Guests: 1)

Top